LSAT and Law School Admissions Forum

Get expert LSAT preparation and law school admissions advice from PowerScore Test Preparation.

 htngo12
  • Posts: 40
  • Joined: May 19, 2016
|
#28444
Hi!

With this strengthen question, I initially thought the argument was set up as:

P1: Introduction new drug into marketplace should be a contingent upon having a good understanding of its social impact.

P2: However, social impact of newly marketed antihistamine is unclear.

Con: There should be a reduction in pace of bringing to the marketplace new drugs that are now tested.

Picked answer (B) social impact of some of the new drugs being tested poorly.

However, when I verified the correct answer (A), I figured the conclusion was actually P1.

So the argument should look like:

P1: Social impact is .. unclear.

Intermediate conclusion: .. reduction in pace of bringing to the market place new drugs are tested.

Con: introduction of ... should .. good understanding..

Then the answer (A) should fit in the argument.

I'm not sure if my reasoning is valid is this question. I just had a hard time trying to figure the logic order of the argument.

Think you can clarify this?
User avatar
 Jonathan Evans
PowerScore Staff
  • PowerScore Staff
  • Posts: 726
  • Joined: Jun 09, 2016
|
#28473
Hi, htngo,

You were correct the first time with the conclusion and the premises. I am unclear what your prephrase/prediction was, but it is clear that you understood the task was to strengthen the conclusion. With that task in mind, let's review briefly what you need to accomplish to strengthen an argument: you need some new information that, if true, would increase the likelihood that the conclusion is valid. This information will address some inherent gap or assumption in the reasoning. You correctly identified as a flaw the fact that we know little about the other drugs coming onto market. However, while this answer choice you chose is germane to this gap, it is actually rather weak tea for a strengthen question. Answer choice B qualifies what we know about the other drugs with the word "some." Watch out for such weak language on strengthen and weaken questions. Since you are seeking to "most" support (or hurt) these arguments, you should prefer strong, definite statements that address the inherent gaps.

Answer choice A also addresses the social impact of other drugs (most other new drugs being tested). The information in answer choice A informs you that the social impact of the antihistamine is in fact greater than that of most of these new drugs being tested. Since the understanding of the social impact of the antihistamine was inadequate according to the premises, this answer choice gives you strong support for the conclusion.

On another note, consider that this question, late in the section, is likely a more difficult LR question. On more difficult LR questions, the answers that seem at first glance most appealing are often wrong. Be on the lookout for answers that seem to match your prephrase verbatim, that seem to be calling out to you, "Pick me! Pick me!" On difficult LR problems, the LSAT writers include such answers to guarantee that a considerable percentage of even strong test-takers choose the wrong answer.
 htngo12
  • Posts: 40
  • Joined: May 19, 2016
|
#28843
Ok,

Now I can see the logic in the argument. I need a strong statement that will piece everything together:

Premises (Social impact unclear and reduction in pace of new drugs now tested)

Answer (Social impact is much better understood)

Conclusion (Introduction of new drug should have a good understanding)

Got it!
 Jay
  • Posts: 46
  • Joined: Jan 09, 2020
|
#85687
Jonathan Evans wrote: Fri Sep 09, 2016 8:41 pm Hi, htngo,

You were correct the first time with the conclusion and the premises. I am unclear what your prephrase/prediction was, but it is clear that you understood the task was to strengthen the conclusion. With that task in mind, let's review briefly what you need to accomplish to strengthen an argument: you need some new information that, if true, would increase the likelihood that the conclusion is valid. This information will address some inherent gap or assumption in the reasoning. You correctly identified as a flaw the fact that we know little about the other drugs coming onto market. However, while this answer choice you chose is germane to this gap, it is actually rather weak tea for a strengthen question. Answer choice B qualifies what we know about the other drugs with the word "some." Watch out for such weak language on strengthen and weaken questions. Since you are seeking to "most" support (or hurt) these arguments, you should prefer strong, definite statements that address the inherent gaps.

Answer choice A also addresses the social impact of other drugs (most other new drugs being tested). The information in answer choice A informs you that the social impact of the antihistamine is in fact greater than that of most of these new drugs being tested. Since the understanding of the social impact of the antihistamine was inadequate according to the premises, this answer choice gives you strong support for the conclusion.

On another note, consider that this question, late in the section, is likely a more difficult LR question. On more difficult LR questions, the answers that seem at first glance most appealing are often wrong. Be on the lookout for answers that seem to match your prephrase verbatim, that seem to be calling out to you, "Pick me! Pick me!" On difficult LR problems, the LSAT writers include such answers to guarantee that a considerable percentage of even strong test-takers choose the wrong answer.
Thank you for your response.

I learned that although the strengthen question stem is phrased "which ... most strengthens the argument" there is only ONE answer that strengthens the argument.

So, if B is not the answer, then B doesn't strengthen the argument. What can I say that B doesn't strengthen the argument?

So, If you see an answer choice like B that is "weak" as you stated above, then what else can you say that it doesn't (completely) strengthen the argument?

Many thanks
User avatar
 Ryan Twomey
PowerScore Staff
  • PowerScore Staff
  • Posts: 141
  • Joined: Mar 04, 2021
|
#85695
There is a lot of debate on whether multiple answer choices can strengthen the argument in logical reasoning. I would personally say that always one answer choice clearly strengthens more than the others. The question stem as you noticed says: "which one of the following, if true, most strengthens the argument," so technically I would say the test makers are asking which one strengthens the most, not that an incorrect answer choice means that it does not strengthen the argument at all.

With regards to explaining answer choice B, I first have to explain the stimulus.

The evidence is two parts. First: we should understand new drugs' social impact before the drugs are released. Second: the new antihistamine's social impact is unknown.

The conclusion is the last sentence saying essentially: we should reduce the pace of drugs to the market that are being tested.

So our assumption is that many other drugs social impact are not understood. We are looking to strengthen that assumption and make that assumption more likely to be true in the answer choices in this particular question.

Answer choice A: makes it so that way most of the drugs social impact is not understood. This is very strong, and much better than I could have ever predicted for a correct answer choice. It is perfect.

Answer choice B: "The social impact of some new drugs being introduced is not understood." Some means at least one. Some is one of the most important words on the test. We already know that we do not understand at least one of the drugs social impact, so this answer choice gives us nothing. It is a restatement of our second premise essentially.

I hope all this helps, and good luck studying!!

Best,
Ryan

Get the most out of your LSAT Prep Plus subscription.

Analyze and track your performance with our Testing and Analytics Package.